Difference between revisions of "2022 AMC 10B Problems/Problem 13"
(→Solution 2) |
(→Solution 2) |
||
Line 19: | Line 19: | ||
The least prime greater than these two primes is <math>79</math> <math>\implies \boxed{\textbf{(E) }16}</math> | The least prime greater than these two primes is <math>79</math> <math>\implies \boxed{\textbf{(E) }16}</math> | ||
+ | |||
~BrandonZhang202415 | ~BrandonZhang202415 |
Revision as of 19:06, 17 November 2022
Solution
Let the two primes be and . We would have and
Solution 2
Let the two primes be and such that and
By the difference of cubes formula,
Plugging in and ,
Through the givens, we can see that .
Thus,
Checking prime pairs near , we find that
The least prime greater than these two primes is
~BrandonZhang202415